Difference between revisions of "2020 AMC 10B Problems/Problem 4"
MRENTHUSIASM (talk | contribs) (Added in Sol 3.) |
|||
Line 18: | Line 18: | ||
Looking at the answer choices, only <math>7</math> and <math>11</math> are coprime to <math>90</math>. Testing <math>7</math>, the smaller angle, makes the other angle <math>83</math> which is prime, therefore our answer is <math>\boxed{\textbf{(D)}\ 7}</math> | Looking at the answer choices, only <math>7</math> and <math>11</math> are coprime to <math>90</math>. Testing <math>7</math>, the smaller angle, makes the other angle <math>83</math> which is prime, therefore our answer is <math>\boxed{\textbf{(D)}\ 7}</math> | ||
+ | |||
+ | ==Solution 3 (Euclidean Algorithm)== | ||
+ | It is clear that <math>\gcd(a,b)=1.</math> By the Euclidean Algorithm, we have <cmath>\gcd(a,b)=\gcd(a+b,b)=\gcd(90,b)=1,</cmath> so <math>90</math> and <math>b</math> are relatively prime. | ||
+ | |||
+ | Since both <math>a</math> and <math>b</math> are prime numbers, the least such value of <math>b</math> is <math>7.</math> We verify that <math>a=90-b=83</math> is also a prime number. Therefore, the answer is <math>\boxed{\textbf{(D)}\ 7}.</math> | ||
+ | |||
+ | ~MRENTHUSIASM | ||
==Video Solution== | ==Video Solution== |
Revision as of 17:04, 18 May 2021
- The following problem is from both the 2020 AMC 10B #4 and 2020 AMC 12B #4, so both problems redirect to this page.
Contents
Problem
The acute angles of a right triangle are and , where and both and are prime numbers. What is the least possible value of ?
Solution 1
Since the three angles of a triangle add up to and one of the angles is because it's a right triangle, .
The greatest prime number less than is . If , then , which is not prime.
The next greatest prime number less than is . If , then , which IS prime, so we have our answer ~quacker88
Solution 2
Looking at the answer choices, only and are coprime to . Testing , the smaller angle, makes the other angle which is prime, therefore our answer is
Solution 3 (Euclidean Algorithm)
It is clear that By the Euclidean Algorithm, we have so and are relatively prime.
Since both and are prime numbers, the least such value of is We verify that is also a prime number. Therefore, the answer is
~MRENTHUSIASM
Video Solution
~IceMatrix
~savannahsolver
~AlexExplains
See Also
2020 AMC 10B (Problems • Answer Key • Resources) | ||
Preceded by Problem 3 |
Followed by Problem 5 | |
1 • 2 • 3 • 4 • 5 • 6 • 7 • 8 • 9 • 10 • 11 • 12 • 13 • 14 • 15 • 16 • 17 • 18 • 19 • 20 • 21 • 22 • 23 • 24 • 25 | ||
All AMC 10 Problems and Solutions |
2020 AMC 12B (Problems • Answer Key • Resources) | |
Preceded by Problem 3 |
Followed by Problem 5 |
1 • 2 • 3 • 4 • 5 • 6 • 7 • 8 • 9 • 10 • 11 • 12 • 13 • 14 • 15 • 16 • 17 • 18 • 19 • 20 • 21 • 22 • 23 • 24 • 25 | |
All AMC 12 Problems and Solutions |
The problems on this page are copyrighted by the Mathematical Association of America's American Mathematics Competitions.